(Created page with "Category:ECE Category:QE Category:problem solving <center> <font size= 4> ECE Ph.D. Qualifying Exam </font size> <font size= 4> Auto...")
 
Line 18: Line 18:
 
----
 
----
 
===Solution===
 
===Solution===
Let <math>t_1=x_1-1</math>, <math>t_2=x_2+1</math><br>
+
Let <math>t_1=x_1-2</math>, <math>t_2=x_2+1</math><br>
so that <math>g(t_1,t_2)=\dfrac{1}{t_1^2+t_2^2+3}</math>
+
so that <math>g(t_1,t_2)=\dfrac{1}{t_1^2+t_2^2+3}|t_1=0,t_2</math> would have some convex property with <math>f(x_1,x_2)=\dfrac{1}{(x_1-2)^2+(x_2+1)^2+3}|x_1=2,x_1=-1</math><br>
 
----
 
----
 
[[QE2016_AC-3_ECE580|Back to QE AC question 3, August 2016]]
 
[[QE2016_AC-3_ECE580|Back to QE AC question 3, August 2016]]
  
 
[[ECE_PhD_Qualifying_Exams|Back to ECE Qualifying Exams (QE) page]]
 
[[ECE_PhD_Qualifying_Exams|Back to ECE Qualifying Exams (QE) page]]

Revision as of 21:59, 18 February 2019


ECE Ph.D. Qualifying Exam

Automatic Control (AC)

Question 3: Optimization

August 2016 Problem 3


Solution

Let $ t_1=x_1-2 $, $ t_2=x_2+1 $
so that $ g(t_1,t_2)=\dfrac{1}{t_1^2+t_2^2+3}|t_1=0,t_2 $ would have some convex property with $ f(x_1,x_2)=\dfrac{1}{(x_1-2)^2+(x_2+1)^2+3}|x_1=2,x_1=-1 $


Back to QE AC question 3, August 2016

Back to ECE Qualifying Exams (QE) page

Alumni Liaison

Basic linear algebra uncovers and clarifies very important geometry and algebra.

Dr. Paul Garrett